Convergence Test for Alternating Series and When to Use It

Click For Summary
The discussion centers on the convergence of the series Ʃ cos(k*pi)/k from 1 to infinity, highlighting the use of the alternating series test. It is noted that the series converges because the terms alternate in sign and decrease in absolute value, with the limit approaching zero. A comparison is made with the series (-1)k(4k/8k), which diverges due to the limit not approaching zero, despite being convergent by the geometric series. Clarifications are sought regarding the application of the comparison test for sine and cosine functions. The conversation emphasizes understanding when to apply the alternating series test and the comparison test effectively.
ichilouch
Messages
9
Reaction score
0

Homework Statement


Ʃ cos(k*pi)/k from 1 to infinity.
This is a test for convergence.

and when is the proper time to use the alternating series test
like using it on (-1)k(4k/8k) would result to divergence
since lim of (4k/8k) is infinity and not 0 but the function is really
convergent by the geometric series?

Homework Equations





The Attempt at a Solution


Is it right to do this:

for k is odd cos is negative and for k is even cos is positive
then
cos(k*pi)\k < (-1)k/k
and by alternating series test ;
(-1)k*(1/k) since 1/k is decreasing and lim as 1/k approaches infinity is 0 then
(-1)k1/k converges thus cos(k*pi)/k converges by direct comparison test.
[
 
Physics news on Phys.org
That is perfectly OK for the alternating 1/k-series! :smile:

"since lim of (4^k/8^k) is infinity"
Is it?
 
Thanks for the reply
So rechecking limit of (4/8)^k as k approaches infinity is 0
Then if it is sine or cosine over a variable, the comparison test for it is (-1)^k?
and if it is sine or cosine squared, the comparison is 1 only?
 
Question: A clock's minute hand has length 4 and its hour hand has length 3. What is the distance between the tips at the moment when it is increasing most rapidly?(Putnam Exam Question) Answer: Making assumption that both the hands moves at constant angular velocities, the answer is ## \sqrt{7} .## But don't you think this assumption is somewhat doubtful and wrong?

Similar threads

  • · Replies 7 ·
Replies
7
Views
2K
  • · Replies 2 ·
Replies
2
Views
1K
  • · Replies 2 ·
Replies
2
Views
2K
  • · Replies 14 ·
Replies
14
Views
2K
  • · Replies 3 ·
Replies
3
Views
1K
  • · Replies 4 ·
Replies
4
Views
1K
  • · Replies 4 ·
Replies
4
Views
1K
  • · Replies 3 ·
Replies
3
Views
1K
  • · Replies 1 ·
Replies
1
Views
2K
Replies
29
Views
2K